Difference between revisions of "2022 AMC 12A Problems/Problem 14"

(Created page with "==Problem== What is the value of <cmath>(\log 5)^{3}+(\log 20)^{3}+(\log 8)(\log 0.25)</cmath> where <math>\log</math> denotes the base-ten logarithm? <math>\textbf{(A)}~\dis...")
 
m (Solution 2)
 
(10 intermediate revisions by 7 users not shown)
Line 2: Line 2:
 
What is the value of <cmath>(\log 5)^{3}+(\log 20)^{3}+(\log 8)(\log 0.25)</cmath> where <math>\log</math> denotes the base-ten logarithm?
 
What is the value of <cmath>(\log 5)^{3}+(\log 20)^{3}+(\log 8)(\log 0.25)</cmath> where <math>\log</math> denotes the base-ten logarithm?
  
<math>\textbf{(A)}~\displaystyle\frac{3}{2}\qquad\textbf{(B)}~\displaystyle\frac{7}{4}\qquad\textbf{(C)}~2\qquad\textbf{(D)}~\displaystyle\frac{9}{4}\qquad\textbf{(E)}~\displaystyle\frac{5}{2}</math>
+
<math>\textbf{(A)}~\frac{3}{2}\qquad\textbf{(B)}~\frac{7}{4}\qquad\textbf{(C)}~2\qquad\textbf{(D)}~\frac{9}{4}\qquad\textbf{(E)}~3</math>
  
==Solution==
+
==Solution 1==
 
Let <math>\text{log } 2 = x</math>. The expression then becomes <cmath>(1+x)^3+(1-x)^3+(3x)(-2x)=\boxed{2}.</cmath>
 
Let <math>\text{log } 2 = x</math>. The expression then becomes <cmath>(1+x)^3+(1-x)^3+(3x)(-2x)=\boxed{2}.</cmath>
  
 
-bluelinfish
 
-bluelinfish
 +
 +
==Solution 2==
 +
Using sum of cubes
 +
<cmath>(\log 5)^{3}+(\log 20)^{3}</cmath>
 +
<cmath>= (\log 5 + \log 20)((\log 5)^{2}-(\log 5)(\log 20) + (\log 20)^{2})</cmath>
 +
<cmath>= 2((\log 5)^{2}-(\log 5)(2\log 2 + \log 5) + (2\log 2 + \log 5)^{2})</cmath>
 +
Let x = <math>\log 5</math> and y = <math>\log 2</math>, so <math>x+y=1</math>
 +
 +
The entire expression becomes
 +
<cmath>2(x^2-x(2y+x)+(2y+x)^2)-6y^2</cmath>
 +
<cmath>=2(x^2+2xy+4y^2-3y^2)</cmath>
 +
<cmath>=2(x+y)^2 = \boxed{2}</cmath>
 +
 +
~kempwood
 +
 +
==Solution 3==
 +
 +
We can estimate the solution. Using <math>\log(2) \approx 0.3, \log(20) = \log(10)+\log(2) = 1 + 0.3 \approx 1.3, \log(8) \approx 0.9</math> and <math>\log(.25) = \log(1)-\log(4)= 0 - 0.6\approx -0.6,</math> we have
 +
 +
<cmath>0.7^3 + 1.7^3 + .9\cdot(-0.6) = \boxed{2}</cmath>
 +
~kxiang
 +
 +
==Solution 4(log bash)==
 +
 +
Using log properties, we combine the terms to make our expression equal to <math>\log {( (5^{\log^2{5}}) \cdot (20^{\log^2{20}}) \cdot 8 ^ {\log{\frac{1}{4}}} ) }</math>.
 +
By exponent properties, we separate the part with base <math>20</math> to become <math>20^{\log^2{5}} \cdot 20^{\log^2{20}-\log^2{5}}</math>. Then, we substitute this into the original expression to get <math>\log {( (5^{\log^2{5}}) \cdot 20^{\log^2{5}} \cdot 20^{\log^2{20}-\log^2{5}} \cdot 8 ^ {\log{\frac{1}{4}}} ) } = \log {( (100^{\log^2{5}}) \cdot 20^{\log^2{20}-\log^2{5}} \cdot 8 ^ {\log{\frac{1}{4}}} ) }</math>.
 +
Because <math>100^{\log^2{5}} = 25^{\log{5}}</math>, and <math>\log^2{20}-\log^2{5} = (\log{20}+\log{5})(\log{20}-\log{5}) = \log{100}\cdot\log{4} = 2\log{4}</math>, this expression is equal to <math> \log {( 25 ^ {\log{5}} \cdot 400^{\log{4}} \cdot 8 ^ {\log{\frac{1}{4}}} ) }</math>.
 +
We perform the step with the base combining on <math>25</math> and <math>400</math> to get <math>25 ^ {\log{5}} \cdot 400^{\log{4}}  = 25 ^ {\log{5}-\log{4}} \cdot 10000^{\log{4}} = 25^{\log{\frac{5}{4}}}\cdot 256</math>. Putting this back into the whole equation gives <math>\log{( 25^{\log{\frac{5}{4}}}\cdot 256 \cdot 8^{\log{\frac{1}{4}}})}</math>.
 +
One last base merge remains - but <math>25\cdot 8</math> isn't a power of 10. We can rectify this by converting <math>8^{\log{\frac{1}{4}}}</math> to <math>(4^\frac{3}{2})^{\log{\frac{1}{4}}} = 4^{\log{ \frac{1}{8} }}</math>.
 +
Finally, we complete this arduous process by performing the base merge on <math>\log{( 25^{\log{\frac{5}{4}}}\cdot 256 \cdot 4^{\log{\frac{1}{8}}})}</math>.
 +
We get <math>25^{\log{\frac{5}{4}}} \cdot 4^{\log{\frac{1}{8}}} = 25^{\log{\frac{5}{4}}-\log{\frac{1}{8}}} \cdot 100^{\log{\frac{1}{8}}} = 25^{\log{10}} \cdot \frac{1}{64} = \frac{25}{64}</math>.
 +
Putting this back into that original equation one last time, we get <math>\log(256 \cdot \frac{25}{64}) = \log{100} = \boxed{2}</math>.
 +
~aop2014
 +
 +
==Video Solution (Speedy)==
 +
https://www.youtube.com/watch?v=pai2A9FXI9U
 +
 +
~Education, the Study of Everything
 +
 +
==Video Solution (Simple)==
 +
https://youtu.be/7yAh4MtJ8a8?si=9vbP5erdxlCLlG82&t=2957
 +
 +
~Math-x
  
 
==See Also==
 
==See Also==
 
{{AMC12 box|year=2022|ab=A|num-b=13|num-a=15}}
 
{{AMC12 box|year=2022|ab=A|num-b=13|num-a=15}}
 
{{MAA Notice}}
 
{{MAA Notice}}

Latest revision as of 21:03, 23 December 2023

Problem

What is the value of \[(\log 5)^{3}+(\log 20)^{3}+(\log 8)(\log 0.25)\] where $\log$ denotes the base-ten logarithm?

$\textbf{(A)}~\frac{3}{2}\qquad\textbf{(B)}~\frac{7}{4}\qquad\textbf{(C)}~2\qquad\textbf{(D)}~\frac{9}{4}\qquad\textbf{(E)}~3$

Solution 1

Let $\text{log } 2 = x$. The expression then becomes \[(1+x)^3+(1-x)^3+(3x)(-2x)=\boxed{2}.\]

-bluelinfish

Solution 2

Using sum of cubes \[(\log 5)^{3}+(\log 20)^{3}\] \[= (\log 5 + \log 20)((\log 5)^{2}-(\log 5)(\log 20) + (\log 20)^{2})\] \[= 2((\log 5)^{2}-(\log 5)(2\log 2 + \log 5) + (2\log 2 + \log 5)^{2})\] Let x = $\log 5$ and y = $\log 2$, so $x+y=1$

The entire expression becomes \[2(x^2-x(2y+x)+(2y+x)^2)-6y^2\] \[=2(x^2+2xy+4y^2-3y^2)\] \[=2(x+y)^2 = \boxed{2}\]

~kempwood

Solution 3

We can estimate the solution. Using $\log(2) \approx 0.3, \log(20) = \log(10)+\log(2) = 1 + 0.3 \approx 1.3, \log(8) \approx 0.9$ and $\log(.25) = \log(1)-\log(4)= 0 - 0.6\approx -0.6,$ we have

\[0.7^3 + 1.7^3 + .9\cdot(-0.6) = \boxed{2}\] ~kxiang

Solution 4(log bash)

Using log properties, we combine the terms to make our expression equal to $\log {( (5^{\log^2{5}}) \cdot (20^{\log^2{20}}) \cdot 8 ^ {\log{\frac{1}{4}}} ) }$. By exponent properties, we separate the part with base $20$ to become $20^{\log^2{5}} \cdot 20^{\log^2{20}-\log^2{5}}$. Then, we substitute this into the original expression to get $\log {( (5^{\log^2{5}}) \cdot 20^{\log^2{5}} \cdot 20^{\log^2{20}-\log^2{5}} \cdot 8 ^ {\log{\frac{1}{4}}} ) } = \log {( (100^{\log^2{5}}) \cdot 20^{\log^2{20}-\log^2{5}} \cdot 8 ^ {\log{\frac{1}{4}}} ) }$. Because $100^{\log^2{5}} = 25^{\log{5}}$, and $\log^2{20}-\log^2{5} = (\log{20}+\log{5})(\log{20}-\log{5}) = \log{100}\cdot\log{4} = 2\log{4}$, this expression is equal to $\log {( 25 ^ {\log{5}} \cdot 400^{\log{4}} \cdot 8 ^ {\log{\frac{1}{4}}} ) }$. We perform the step with the base combining on $25$ and $400$ to get $25 ^ {\log{5}} \cdot 400^{\log{4}}  = 25 ^ {\log{5}-\log{4}} \cdot 10000^{\log{4}} = 25^{\log{\frac{5}{4}}}\cdot 256$. Putting this back into the whole equation gives $\log{( 25^{\log{\frac{5}{4}}}\cdot 256 \cdot 8^{\log{\frac{1}{4}}})}$. One last base merge remains - but $25\cdot 8$ isn't a power of 10. We can rectify this by converting $8^{\log{\frac{1}{4}}}$ to $(4^\frac{3}{2})^{\log{\frac{1}{4}}} = 4^{\log{ \frac{1}{8} }}$. Finally, we complete this arduous process by performing the base merge on $\log{( 25^{\log{\frac{5}{4}}}\cdot 256 \cdot 4^{\log{\frac{1}{8}}})}$. We get $25^{\log{\frac{5}{4}}} \cdot 4^{\log{\frac{1}{8}}} = 25^{\log{\frac{5}{4}}-\log{\frac{1}{8}}} \cdot 100^{\log{\frac{1}{8}}} = 25^{\log{10}} \cdot \frac{1}{64} = \frac{25}{64}$. Putting this back into that original equation one last time, we get $\log(256 \cdot \frac{25}{64}) = \log{100} = \boxed{2}$. ~aop2014

Video Solution (Speedy)

https://www.youtube.com/watch?v=pai2A9FXI9U

~Education, the Study of Everything

Video Solution (Simple)

https://youtu.be/7yAh4MtJ8a8?si=9vbP5erdxlCLlG82&t=2957

~Math-x

See Also

2022 AMC 12A (ProblemsAnswer KeyResources)
Preceded by
Problem 13
Followed by
Problem 15
1 2 3 4 5 6 7 8 9 10 11 12 13 14 15 16 17 18 19 20 21 22 23 24 25
All AMC 12 Problems and Solutions

The problems on this page are copyrighted by the Mathematical Association of America's American Mathematics Competitions. AMC logo.png